Using logarithms in vector Calculus

Click For Summary
The discussion focuses on the derivation of equation (3) from equations (1) and (2) in vector calculus involving logarithms. The user is confused about how the log term from equation (2) is incorporated into equation (3) and questions the cancellation of terms. They initially suggest that identifying 1/n as ∇L raises questions about the disappearance of ∇n in the derivation. Ultimately, they conclude that the mentor likely applied the multivariable chain rule to transform ∇n*1/n into ∇L. The conversation highlights the complexities of applying logarithmic identities in vector calculus.
Mzzed
Messages
67
Reaction score
5

Homework Statement


My mentor has run me through the derivation of equation (3) bellow. I am unsure how he went from (1) to (3) by incorporating the log term from eq(2). In eq(3) it seems he just canceled the relevant n terms and then identified 1/n as the derivative of L however if this were the case then I am unsure why only one of the two terms on the RHS of eq(3) contains the L term.

How did he get from (1) to (3) using eq(2)?

Homework Equations


1) (∇⋅ωn)/n = (n∇⋅ω + ω⋅∇n)/n
2) L = log(n)
3) ∇⋅ω = ∇⋅ω + ω⋅∇L
where w is a 3 dimensional vector and n is a scalar.

The Attempt at a Solution


I think he may have just identified 1/n = ∇L but then if this were true, where did ∇n go from the second term in eq(1)? Also if this were true the same ∇L term would then be found in the first term wouldn't it?
 
Physics news on Phys.org
Your equation 3 is missing stuff on the left-hand side.

In general for a logarithm you will have ##d\log(f) = df/f##.
 
I finally realized he used the multivariable chain rule to turn ∇n*1/n into ∇L. Thanks for your help though!
 
  • Like
Likes Delta2
Question: A clock's minute hand has length 4 and its hour hand has length 3. What is the distance between the tips at the moment when it is increasing most rapidly?(Putnam Exam Question) Answer: Making assumption that both the hands moves at constant angular velocities, the answer is ## \sqrt{7} .## But don't you think this assumption is somewhat doubtful and wrong?

Similar threads

  • · Replies 3 ·
Replies
3
Views
3K
Replies
11
Views
2K
  • · Replies 6 ·
Replies
6
Views
2K
  • · Replies 1 ·
Replies
1
Views
3K
  • · Replies 7 ·
Replies
7
Views
1K
  • · Replies 11 ·
Replies
11
Views
2K
Replies
5
Views
2K
  • · Replies 4 ·
Replies
4
Views
1K
  • · Replies 1 ·
Replies
1
Views
2K
  • · Replies 11 ·
Replies
11
Views
2K